Curvature of polar function r=4cos(3θ): Find Solution

  • Thread starter Thread starter meson0731
  • Start date Start date
  • Tags Tags
    Curvature
Click For Summary
To find the curvature of the polar function r = 4cos(3θ), the user converted it to parametric form, resulting in x = 4cos(3θ)cos(θ) and y = 4cos(3θ)sin(θ). They calculated the first and second derivatives to obtain the velocity and acceleration vectors, respectively. The user expressed concern about the complexity of computing the cross product for curvature and sought advice on simplification or alternative methods. Suggestions included using standard trigonometric identities to simplify the parametric equations and referring to resources for the polar formulation of curvature. The discussion emphasizes the importance of trigonometric relations in solving the problem efficiently.
meson0731
Messages
13
Reaction score
0

Homework Statement


Given the polar function r = 4cos(3θ) find the curvature.



Homework Equations





The Attempt at a Solution



I know there is a formula for curvature of a polar function but I was never given that equation and was told to convert to parametric. and use ||v x a|| / ||v||^3. So this is my work
Converting to parametric:

x = 4cos(3θ)cos(θ)
y= 4cos(3θ)sin(θ)
z = 0

to get v i took the derivative

x' = -12sin(3θ)cos(θ) - 4cos(3θ)sin(θ)
y' = -12sin(3θ)sin(θ) + 4 cos(4θ)cos(θ)
z' = 0

I then took the second derivative to get a

x'' = -40cos(3θ)cos(θ) + 24sin(3θ)sin(θ)
y'' = -40cos(3θ)sin(θ) - 24sin(3θ)cos(θ)
z'' = 0

However I feel like i did something wrong because to computer ||v x a|| would be really messy. Is there any way to simplify before I do the cross product or is there is easier way and I'm just doing the problem wrong? Thanks any help is appreciated.
 
Physics news on Phys.org
Question: A clock's minute hand has length 4 and its hour hand has length 3. What is the distance between the tips at the moment when it is increasing most rapidly?(Putnam Exam Question) Answer: Making assumption that both the hands moves at constant angular velocities, the answer is ## \sqrt{7} .## But don't you think this assumption is somewhat doubtful and wrong?

Similar threads

  • · Replies 4 ·
Replies
4
Views
2K
Replies
3
Views
1K
  • · Replies 2 ·
Replies
2
Views
2K
Replies
7
Views
2K
  • · Replies 1 ·
Replies
1
Views
2K
  • · Replies 1 ·
Replies
1
Views
2K
  • · Replies 1 ·
Replies
1
Views
2K
  • · Replies 7 ·
Replies
7
Views
8K
  • · Replies 6 ·
Replies
6
Views
2K